suma szeregu

Własności ciągów i zbieżność, obliczanie granic. Twierdzenia o zbieżności.
mateuszwdowiak
Użytkownik
Użytkownik
Posty: 20
Rejestracja: 19 lut 2007, o 23:48
Płeć: Mężczyzna
Lokalizacja: grodzisk maz
Podziękował: 2 razy
Pomógł: 1 raz

suma szeregu

Post autor: mateuszwdowiak »

Witam, mam taki problem:
Jak udowodnić, że:
\(\displaystyle{ \sum_{n=1}^{ } ft(-1 \right) ^{n+1} \frac{1}{n} = ln 2}\)
nie używając wielomianów Taylora?
Awatar użytkownika
LecHu :)
Użytkownik
Użytkownik
Posty: 953
Rejestracja: 23 gru 2005, o 23:46
Płeć: Mężczyzna
Lokalizacja: BFGD
Podziękował: 16 razy
Pomógł: 162 razy

suma szeregu

Post autor: LecHu :) »

\(\displaystyle{ \sum_{n=1}^{ }\left(-1 \right)^{n+1}\frac{1}{n}=\sum_{n=1}^{ }\frac{1}{n(n+1)}}\)
Może to pomoże.
mateuszwdowiak
Użytkownik
Użytkownik
Posty: 20
Rejestracja: 19 lut 2007, o 23:48
Płeć: Mężczyzna
Lokalizacja: grodzisk maz
Podziękował: 2 razy
Pomógł: 1 raz

suma szeregu

Post autor: mateuszwdowiak »

skad to wziales, to jest nieprawda.
Awatar użytkownika
LecHu :)
Użytkownik
Użytkownik
Posty: 953
Rejestracja: 23 gru 2005, o 23:46
Płeć: Mężczyzna
Lokalizacja: BFGD
Podziękował: 16 razy
Pomógł: 162 razy

suma szeregu

Post autor: LecHu :) »

Rozpatrz sobie to dla parzystych i nieparzystych.
mateuszwdowiak
Użytkownik
Użytkownik
Posty: 20
Rejestracja: 19 lut 2007, o 23:48
Płeć: Mężczyzna
Lokalizacja: grodzisk maz
Podziękował: 2 razy
Pomógł: 1 raz

suma szeregu

Post autor: mateuszwdowiak »

jak wezmiesz tylko wyrazy nieparzyste z tego szeregu, wtedy to bedzie prawda. ale to nic sie z tego nie robi, jest po prostu ten szereg co byl na poczatku.

[ Dodano: 16 Grudnia 2007, 20:54 ]
powiedz mi, po jakich przeksztalceniach ci wychodzi ln2, bo mi w zaden sposob nie chce..
Awatar użytkownika
LecHu :)
Użytkownik
Użytkownik
Posty: 953
Rejestracja: 23 gru 2005, o 23:46
Płeć: Mężczyzna
Lokalizacja: BFGD
Podziękował: 16 razy
Pomógł: 162 razy

suma szeregu

Post autor: LecHu :) »

Rozpisuje sobie kilka pierwszych wyrazów:
\(\displaystyle{ ...=1-\frac{1}{2}+\frac{1}{3}-\frac{1}{4}+\frac{1}{2k+1}-\frac{1}{2k+2}+...=}\)
\(\displaystyle{ =\frac{2-1}{1{\cdot}2}+\frac{4-3}{3{\cdot}4}+...+\frac{2k+2-(2k+1)}{(2k+1){\cdot}(2k+2)}+...}\)
Szczerze mówiąc szeregi doprowadzają mnie do bólu głowy Zcałkowałem to coś i wyszedł ln2.
mateuszwdowiak
Użytkownik
Użytkownik
Posty: 20
Rejestracja: 19 lut 2007, o 23:48
Płeć: Mężczyzna
Lokalizacja: grodzisk maz
Podziękował: 2 razy
Pomógł: 1 raz

suma szeregu

Post autor: mateuszwdowiak »

nie widze tego..
Awatar użytkownika
LecHu :)
Użytkownik
Użytkownik
Posty: 953
Rejestracja: 23 gru 2005, o 23:46
Płeć: Mężczyzna
Lokalizacja: BFGD
Podziękował: 16 razy
Pomógł: 162 razy

suma szeregu

Post autor: LecHu :) »

Czego?Korzystam z tego, że:
\(\displaystyle{ \frac{1}{k(k+1)}=\frac{1}{k}-\frac{1}{k+1}}\)
mateuszwdowiak
Użytkownik
Użytkownik
Posty: 20
Rejestracja: 19 lut 2007, o 23:48
Płeć: Mężczyzna
Lokalizacja: grodzisk maz
Podziękował: 2 razy
Pomógł: 1 raz

suma szeregu

Post autor: mateuszwdowiak »

no to jest oczywiste. jak wiele innych przeksztalcen ktorych sobie mozna tutaj zrobic. ale przede wszystkim wzor masz nieprawdziwy, a poza tym nie widze, zeby w jakikolwiek sposob przyblizal sie do wyniku.
Awatar użytkownika
LecHu :)
Użytkownik
Użytkownik
Posty: 953
Rejestracja: 23 gru 2005, o 23:46
Płeć: Mężczyzna
Lokalizacja: BFGD
Podziękował: 16 razy
Pomógł: 162 razy

suma szeregu

Post autor: LecHu :) »

Zapis jest zły to fakt.
mateuszwdowiak
Użytkownik
Użytkownik
Posty: 20
Rejestracja: 19 lut 2007, o 23:48
Płeć: Mężczyzna
Lokalizacja: grodzisk maz
Podziękował: 2 razy
Pomógł: 1 raz

suma szeregu

Post autor: mateuszwdowiak »

trzeba zastosować stałą eulera, że 1+1/2+1/3+...+1/n-lnn=gamma.... tylko co z tego?

[ Dodano: 18 Grudnia 2007, 23:29 ]
\(\displaystyle{ \lim_{n \to } 1+ \frac{1}{2} + \frac{1}{3} + \frac{1}{4} + + \frac{1}{n} -lnn=\gamma

\lim_{n \to } 1+ \frac{1}{2} + \frac{1}{3} + \frac{1}{4} + + \frac{1}{n}+ \frac{1}{n+1} + + \frac{1}{2n} -ln2n=\gamma }\)

\(\displaystyle{ 1 + \frac{1}{2} +\frac{1}{3} + \frac{1}{4} + + \frac{1}{n}+ \frac{1}{n+1} + + \frac{1}{2n} -ln2n- ft[1+ \frac{1}{2} + \frac{1}{3} + \frac{1}{4} + + \frac{1}{n} -lnn \right] \gamma-\gamma=0
}\)

\(\displaystyle{ 1 + \frac{1}{2} +\frac{1}{3} + \frac{1}{4} + + \frac{1}{n}+ \frac{1}{n+1} + + \frac{1}{2n} - ft[1+ \frac{1}{2} + \frac{1}{3} + \frac{1}{4} + + \frac{1}{n} \right] ln2n-lnn=ln2=1+ \frac{1}{2}-1+ \frac{1}{3} + \frac{1}{4}- \frac{1}{2} + \frac{1}{5} + = \sum_{n=1}^{ } \frac{ ft(-1 \right) ^{n+1} }{n}}\)
ODPOWIEDZ